0 Daumen
4,9k Aufrufe

Hi allerseits,

folgendes Problem:

\( \lim\limits_{x\to\infty} \) (1+ 1/x2 )x = a

 
Meine Frage dazu wäre, ob ich argumentieren darf, dass

a = \( \lim\limits_{x\to\infty} \) (\( \lim\limits_{x\to\infty} \) (1 + 1/x2))x   (also Produktregel der Grenzwertsätze?)

oder ob ich eine Abschätzung via Sandwich-Lemma vornehmen muss.

Vielen danke für jede hilfe :)

Avatar von

Versuch mal das Wurzelkriterium

Tipp: Verwende

https://de.wikipedia.org/wiki/Eulersche_Zahl#Definition

Skärmavbild 2019-01-24 kl. 20.59.20.png

Substituiere x^2 = t

Dann hast du im Exponenten √(t) .

Warum hast du n in der Überschrift und x in der Frage?

Kommt an sich nicht drauf an.

2 Antworten

+1 Daumen

Also ich hoffe, dass ich jetzt nicht was komplett falsches dir antworte:)


$$lim_{x \rightarrow \infty} (1 + \frac{1}{x^2})^x \\ =  lim_{x \rightarrow \infty} (1 + \frac{1}{x^2}) \cdot lim_{x \rightarrow \infty} (1 + \frac{1}{x^2}) \cdot lim_{x \rightarrow \infty} (1 + \frac{1}{x^2}) \cdot \dots  \cdot lim_{x \rightarrow \infty} (1 + \frac{1}{x^2}) \text{ das x-Mal} \\ = 1 \cdot 1 \cdot 1 \cdot \dots \cdot 1 = 1$$

Avatar von

So habe ich mir das auch gedacht, meine Frage ist nur ob das Formal korrekt ist.

Ja, das ist korrekt.Man darf bei

lim f(x) * g(x) = lim f(x) * lim g(x) machen:)

Das geht nicht, denn nach deiner Begründung wäre auch

lim x---> oo  (1+1/x)^x =1 , aber der Grenzwert gibt bekanntlich e.

Die Regel lim f(x)^n

=  (lim f(x))^n gilt nur für konstante n und nicht wenn n=n(x) eine Funktion von x ist. Dann wird es komplizierter.

Könntest du dann auch mal deine Regel

\( \lim\limits_{a\to\infty} \) ( \( \lim\limits_{b\to\infty} \) ( (f(b))g(b) )h(a)  =  \( \lim\limits_{c\to\infty} \) ( f(c) )( g(c)·h(c) ) 

begründen ?

Das hab ich doch nirgends verwendet ?! Das = ist eine elementare Umformung nach Potenzgesetz, sodass man sieht, dass die neu entstandene Basis gegen eine Zahl ungleich 1 konvergiert. Also kann man abschätzen.

Das hab ich doch nirgends verwendet ?

Doch !  Nämlich mit  f(x) = 1+1/x^2 ,  g(x) = x^2  und  h(x) = 1/x

Aber bevor du dich an einem Beweis versuchst :  Es funktioniert z.B. nicht für
f(x) = e ,  g(x) = -x ,  h(x) = 1/x

Dein Trivialbeispiel verstehe ich nicht. Ich habe bereits gesagt, dass ich deine Regel nicht verwendet habe. Es handelt sich um eine Termumformung, da wird der Grenzwert noch gar nicht betrachtet. Und dann wird abgeschätzt.

+1 Daumen

(1+1/x^2)^x

=((1+1/x^2)^(x^2))^(1/x)

Der innere Term konvergiert gegen e, und da außen noch die xte Wurzel genommen wird strebt alles gegen 1.

Avatar von 37 k

Ein anderes Problem?

Stell deine Frage

Willkommen bei der Mathelounge! Stell deine Frage einfach und kostenlos

x
Made by a lovely community